You are on page 1of 100

GMAT-MATH

1. If x , then = =
 
(A)3x+7
(B)3x-7
(C)3x+1
(D)x+7
(E)x-7
1. If x=2,

13
A is the correct answer
2. What is the value of x?
(1)x3 = 8
(2)x2 = 4
 
(A) Statement (1) ALONE is sufficient, but statement (2) alone is not
sufficient.
(B) Statement (2) ALONE is sufficient, but statement (1) alone is not
sufficient.
(C)BOTH statements TOGETHER are sufficient, but NEITHER statement
ALONE is sufficient.
(D)EACH statement ALONE is sufficient.
2. A
= 8, then x = 2
Statement (1) is sufficient, Write down AD.

= 4, x could be 2 or -2
Statement (2) is therefore insufficient.

(A) is the correct answer.


3. If 2, then =

(A)
3. 2 = , =
Thus y =

= = = =

The correct answer is (C).


4. At apartment complex Z, 30 percent of the residents are men over the age of
18, and 40 percent are women over the age of 18. If there are 24 children
living in the complex, how many total residents live in the apartment complex
Z?
 
(A)32
(B)80
(C)94
(D)112
(E)124
 
4.

The answer is (B).


5. If ax + ay + az = 15, what is x + y + z?
(1) x = 2
(2) a = 5
 
(A)Statement (1) ALONE is sufficient, but statement (2) alone is not sufficient.
(B) Statement (2) ALONE is sufficient, but statement (1) alone is not sufficient.
(C)BOTH statements TOGETHER are sufficient, but NEITHER statement ALONE is
sufficient.
(D)EACH statement ALONE is sufficient.
(E)Statements (1) and (2) TOGETHER are not sufficient.
 
5. B
6.
(A) 0.96
(B)0.333
(C)3.0
(D)30.0
(E)96.0
6. C
= , which yields
= , which reduces to = 3
7. The average (arithmetic mean) of five numbers is 7. If the
average of three of the numbers is 5, then what is the average of the
other two numbers?
 
(A)5
(B)10
(C)12
(D)15
(E)20
7. B

Average of 5 numbers is 7, numbers must add up to 35


Three of those number average 5, numbers must add up 15
The sum of two numbers left is 35-15 = 20
Average of those two numbers is 10
8. If $5,000 is invested in an account that earns 8% interest compounded semi-
annually, then the interest earned after one year would be how much greater
than if the $5,000 had been invested at 8% simple yearly interest?
(A)$4
(B)$8
(C)$12
(D)$16
(E)$432
8. B
Determine two amounts of interest
4% of 5,000 is 200.
4% of the new balance $5,200 is 208, and the account contains $5,408

For the other account, 8% of 5,000 is 400, so the account contains $5,400

The difference is $5,408 - $5,400 = $8


9. If x > 0 and x percent of a is b, then, in terms of x, a is how
many times b?
 
(A)100x
(B)10x
(C)x

(D)
9. E
Plug in.
Let x = 20 and a = 100.
20% of 100 is 20, so b=20
100 is 5 times 20, so the target is 5.
Now, Plug In x=20 into the answer choices looking for the target 5.
Only (E) matches
10. If Teena is driving at 55 miles per hour and is currently 7.5 miles behind
Joe, who is driving at 40 miles per hour in the same direction, then in how
many minutes will Teena be 15 miles ahead of Joe?
 
(A)15
(B)60
(C)75
(D)90
(E)105
10. D

Difference is 55 – 40 = 15 miles per hour


Relative distance is 15 + 7.5 miles = 22.5 miles
D = RT, and solve for T. 22.5 = 15T yields that T = , or T = 1.5 hours.
1.5 60 = 90 minutes
11. In the figure above, if l1 l2, which of the following angles must
be equivalent?
 
(A)a and b
(B)g and f
(C)d and e
(D)a and d
(E)f and d
 
11. D

Both a and d are big angles created as line across and and
therefore a and d must be equal
12. In the diagram above, if the area of triangle LNO is 32, then
what is the area of triangle LMN?

(A)24
(B)24
(C)24
(D)32
(E)48
12. B

The area of a triangle is bh, so 32 = (LO)8, which simplifies to 32


= 4LO, and LO = 8.
LNO is 45-45-90 triangle, x: x: x
LN has a length of 8
LN as the base of triangle LMN, height is 6
Thus, bh = (8)6, which simplifies to 3(8) = 24
13. In the figure above, P is the center of the larger circle, and N is the center
of the smaller, shaded circle. If the radius of the smaller circle is 5, what is the
area of the unshaded region.
 
(A)100𝜋
(B)75𝜋
(C)25𝜋
(D)20𝜋
(E)10𝜋
 
13. B

Radius of the circle with center N is 5. So, the area is 𝜋 = 𝜋(25𝜋

Radius of the circle with center P is 2 5 = 10. The area is 𝜋 = 𝜋(

The area of the unshaded region is - 25𝜋 = 75


14. Luxo paint contains only alcohol and pigment. What is the ratio of alcohol to pigment in
Luxo paint?
(1) Exactly 7 ounces of pigments are contained in a 12-ounce can of Luxo paint.
(2) Exactly 5 ounces of alcohol are contained in a 12-ounce can of Luxo paint.
 
(A)Statement (1) ALONE is sufficient, but statement (2) alone is not sufficient.
(B) Statement (2) ALONE is sufficient, but statement (1) alone is not sufficient.
(C)BOTH statements TOGETHER are sufficient, but NEITHER statement ALONE is
sufficient.
(D)EACH statement ALONE is sufficient.
(E)Statements (1) and (2) TOGETHER are not sufficient.
 
 
14. D

 
 
15.If x is a positive number, is x < 1?
 
(1) 2x < 1
(2) 2x ⦤ 2
 
(A)Statement (1) ALONE is sufficient, but statement (2) alone is not sufficient.
(B) Statement (2) ALONE is sufficient, but statement (1) alone is not sufficient.
(C)BOTH statements TOGETHER are sufficient, but NEITHER statement ALONE is
sufficient.
(D)EACH statement ALONE is sufficient.
(E)Statements (1) and (2) TOGETHER are not sufficient.
 
15. A 
16. If x is an integer, is y an integer?

(1) The average (arithmetic mean) of x, y, and y-2 is x.

(2) The average (arithmetic mean) of x and y is not an integer.


(A)Statement (1) ALONE is sufficient, but statement (2) alone is not
sufficient.
(B) Statement (2) ALONE is sufficient, but statement (1) alone is not
sufficient.
(C)BOTH statements TOGETHER are sufficient, but NEITHER statement
ALONE is sufficient.
(D)EACH statement ALONE is sufficient.
(E)Statement (1) and (2) TOGETHER are not sufficient.
16. From statement (1), = x simplifies to y = x + 1. Since x is an
integer, x and y are consecutive integers. Thus, (1) alone is sufficient, and the
answer must be A or D.

From statement (2) alone, y might be an integer (e.g., x = 5 and y =6) or y


might not be an integer (e.g., x =5 and y = 6.2). In both examples, the average
is not an integer. Therefore, (2) alone is not sufficient, and the best answer is
A.
17. If x = 6 what is the value of ?
(A)-30
(B)6
(C)8
(D)30
(E)It cannot be determined from the information given.
If x = 6, then
=
=
=6

(B) is the correct answer.


18. If x-5 = 3x + 2, then x=
(A) -8
(B) -7
(C) -
(D)
(E)
18. (C) is the correct answer.
19. If x=6 what is the value of ?
(A)-30
(B)6
(C)8
(D)30
(E)It cannot be determined from the information given.
90
 
19. B 
20. Sam and Jessica are invited to a dance. If there are 7 men and 7 women in total at the
dance, and one woman and one man are chosen to lead the dance, what is the probability that
Sam and Jessica will NOT be the pair chosen to lead the dance?
 
(A)
(B)
(C)
(D)
(E)
20. E

P(Sam will be chosen) =

P(Jessica will be chosen) =


P(Both will be chosen) = =

P(An event does NOT happen) = 1 =


21. The number of flights leaving a certain airport doubles during every one-
hour period between 9 A.M. and noon; after noon, the number of flights
leaving from the airport doubles during every two-hour period. If 4 flights left
from the airport between 9 and 10 A. M., how many flights left the airport
between 2 and 4 P. M. ?
 
(A)32
(B)48
(C)64
(D)128
(E)256
21. C

9 and 10 am..........4 flights left


10 and 11 am.........8 flights left
11 and noon..........16 flights left
noon and 2pm……32 flights left
2pm and 4pm……64 flights left
22. The ratio to is equal to
(A) to 2
(B) 24 to 7
(C)14 to 9
(D)9 to 14
(E)7 to 24
22. D

Factor as = , which simplifies to =


Second Expression =
22. D
= , which yields = , and reduces to

= , or 9 to 14 ratio
23.List I: {y, 2,4,7,10,11} List II:
{3,3,4,6,7,10}
 
If the median of List I is equal to the sum of the median of list II and the
mode of List II, then y equals
(A)5
(B)7
(C)8
(D)9
(E)10
23. D

List II, the mode is 3 and median is 5. Their sum is 8.


Median of List 1 must also be 8. There are 6 elements in
List I. Median will be the average of 3 rd and 4th numbers.
Those two numbers must add up to 16.
Since 7 is already listed, to get a median of 8 is if y is 9.
24. 5 years ago, Joe was twice as old as Jessica. If in 4 years Joe
will be one and a half times as old as Jessica, then how old is Joe
now?
(A)23
(B)25
(C)27
(D)29
(E)31
24. A
25. At a local office, each trainee can stuff as many envelopes per day as a full time worker.
If there are as many trainees as there are full time workers, then what fraction of all the
envelopes stuffed in a day did the trainees stuff?
(A)
(B)

(C)
25. C
Let the number of full-time workers be 5. There are as many as trainee as
there are full-time workers, so there are 5 = 2 trainees
Let the number of envelops each full-time worker can stuff per day be 3.
So each trainee can stuff 3 = 2 envelops per day
Full-time workers can stuff 5 3 = 15 envelops, trainees stuff 2 2 = 4
envelopes
Two groups stuffs 15 4 = 19 envelops
Trainee stuff 4 envelops, so they stuffed of the envelops.
26. If Set X contains 10 consecutive integers and the sum of the 5
least members of the set is 265, then what is the sum of the 5
greatest members of the set?
 
(A)290
(B)285
(C)280
(D)275
(E)270
26. A

We assign x to represent the smallest integer, then x + (x+1) +


(x+2) + (x+3) + (x+4) =265, so 5x + 10 = 265, 5x = 255, and
x = 51. The five least consecutive integers are thus 51 through
55.
The five greatest integers are 56 through 60. The total is 290.
27. If only people who paid deposits attended the Rose Seminar, how many people attended
this year?
(1) 70 people sent in deposit to attend the Rose Seminar this year.
(2) 60% of the people who sent deposits to attend the Rose Seminar this year actually went.
(A)Statement (1) ALONE is sufficient, but statement (2) alone is not sufficient.
(B)Statement (2) ALONE is sufficient, but statement (1) alone is not sufficient.
(C)BOTH statements TOGETHER are sufficient, but NEITHER statement ALONE is
sufficient.
(D)EACH statement ALONE is sufficient.
(E)Statements (1) and (2) TOGETHER are not sufficient.
27. C
28. =
(A)
(B)
(C)
(D) 1
(E)
28. C
29. What is the value of xy?
(1)x2 = 4
(2)y2 = 9
(A)Statement (1) ALONE is sufficient, but statement (2) alone is not
sufficient.
(B)Statement (2) ALONE is sufficient, but statement (1) alone is not
sufficient.
(C)BOTH statements TOGETHER are sufficient, but NEITHER statement
ALONE is sufficient.
(D)EACH statement ALONE is sufficient.
(E)Statements (1) and (2) TOGETHER are not sufficient.
29. E
30. Are x and y integers?
(1) The product xy is an integer.
(2) x +y is an integer.
 
(A)Statement (1) ALONE is sufficient, but statement (2) alone is not sufficient.
(B)Statement (2) ALONE is sufficient, but statement (1) alone is not sufficient.
(C)BOTH statements TOGETHER are sufficient, but NEITHER statement ALONE is
sufficient.
(D)EACH statement ALONE is sufficient.
(E)Statements (1) and (2) TOGETHER are not sufficient.
30. E
a

𝑏°
d O
c
31. In the circle above, center O is intersected by 2 straight lines,
and 3a = b. What is value of b-a?
 
(A)2
(B)30
(C)45
(D)90
(E)135
 
31. D

a + b =180 a + b =180, and a = 45, b


3a = b must equal 135
a + b = 180 b-a = 135-45, the answer i
a + 3a =180 is 90.
4a = 180
a =45
32. What is the surface area of rectangular solid Y?
(1) The dimensions of one face of rectangular solid Y are 2 by 3.
(2) The volume of rectangular solid Y is 12.
 
(A)Statement (1) ALONE is sufficient, but statement (2) alone is not sufficient.
(B)Statement (2) ALONE is sufficient, but statement (1) alone is not sufficient.
(C)BOTH statements TOGETHER are sufficient, but NEITHER statement ALONE is
sufficient.
(D)EACH statement ALONE is sufficient.
(E)Statements (1) and (2) TOGETHER are not sufficient.
32. C
33. If x is a positive integer, is the greatest common factor of 150 and x a prime number?
 
(1) x is a prime number.
(2) x < 4
 
(A)Statement (1) ALONE is sufficient, but statement (2) alone is not sufficient.
(B) Statement (2) ALONE is sufficient, but statement (1) alone is not sufficient.
(C)BOTH statements TOGETHER are sufficient, but NEITHER statement ALONE is
sufficient.
(D)EACH statement ALONE is sufficient.
(E)Statements (1) and (2) TOGETHER are not sufficient.
33. C
34. If (16)(32) = x(23), then x =
 
(A)81
(B)72
(C)18
(D)16
(E)8
 
34. C
35. What percent of 112 is 14?
 
(A)0.125%
(B)8%
(C)12.5%
(D)125%
(E)800%
35. C
36. The new recruits of a military organization who score in the bottom 16 percent on their
physical conditioning tests are required to retest. If the scores are normally distributed and have
an arithmetic mean of 72, what is the score at or below which the recruits are required to retest?
 
(1) There are 500 new recruits.
(2) 10 new recruits score at least 82 on the physical conditioning test.
 
(A)Statement (1) ALONE is sufficient, but statement (2) alone is not sufficient.
(B)Statement (2) ALONE is sufficient, but statement (1) alone is not sufficient.
(C)BOTH statements TOGETHER are sufficient, but NEITHER statement ALONE is sufficient.
(D)EACH statement ALONE is sufficient.
(E)Statements (1) and (2) TOGETHER are not sufficient.
36. C
37. A glucose solution contains 15 grams of glucose per 100 cubic centimeters
of solution. If 45 cubic centimeters of the solution were poured into an empty
container, how many grams of glucose would be in the container?
 
(A)3.00
(B)5.00
(C)5.50
(D)6.50
(E)6.75
37. E

If x is the number of grams of glucose in the 45 cubic


centimeters of solution, then = , and x = 6.75.
38. What is the ratio of to ?
 

(A)
(B)
(C)

 
38. C
39. A worker is digging a ditch. He gets 2 assistants who work as fast as he
does. If all 3 work on a ditch they should finish it in what fraction of the time
that the worker takes working alone?

(A)
39. A

Since each assistant does as much as the worker, all 3 will


accomplish 1 + 2 or as much as the worker by himself.
So they will finish the job in 1 + or as much as it would
take the worker by himself.
40. If a and b are positive integers, is ab odd?
 
(1) b = 3
(2) a and b are consecutive integers.
 

(A)Statement (1) ALONE is sufficient, but statement (2) alone is not sufficient.
(B) Statement (2) ALONE is sufficient, but statement (1) alone is not sufficient.
(C)BOTH statements TOGETHER are sufficient, but NEITHER statement ALONE is
sufficient.
(D)EACH statement ALONE is sufficient.
(E)Statements (1) and (2) TOGETHER are not sufficient.
40. B
41. What is the quotient when 0.25% of 600 is divided by 0.25 of
600?
 
(A)10
(B)1
(C)0.1
(D)0.01
(E)0.001
41. D

0.25 600 = 150


0.25% can be represented as , or 0.0025
When you multiply by 600, the result is 1.5.
The quotient is thus 1.5 divided by 150, or 0.01.
42. A first-grade teacher uses ten flash cards, numbered 1 through 10, to teach
her students to order numbers correctly. She has students choose four flash
cards at random and arrange the cards in ascending order, if she removes the
cards number 2 and 4, how many different correctly ordered arrangements of
the four selected cards are possible?
 
(A)70
(B)210
(C)336
(D)840
42. A
Because the teacher has removed two cards from the deck,
only 8 cards remain. Make four spots for the cards chosen,
then fill in the number of the options for each spot: 8 7 6
5. Then, divided by the number of ways to arrange these
four cards: 4 3 2 1. Reduce and solve to find 70. The
correct answer is (A).
43. Working at a constant rate, Sam can finish a job in 3 hours. Mark, also
working at a constant rate, can finish the same job in 12 hours. If they work
together for 2 hours, how many minutes will it take Sam to finish the job,
working alone at his constant rate?
 
(A)5
(B)20
(C)30
(D)60
(E)120
 
43. C
APPLIED ARITHMETIC
(CONTD.)
Sam Mark

1 1

1
1 3 12
12
3
APPLIED ARITHMETIC
(CONTD.)
A New Pie

10
12 2= =

5 2
12
APPLIED ARITHMETIC
(CONTD.)
Last Work Pie

1
6 =

11
3 2
44. Three basketball teams play in a league against each other. At the end of
the season, how many different ways could the 3 teams end up ranked against
each other?
 
(A)1
(B)3
(C)6
(D)36
(E)72
44. C

ABC ACB BAC BCA CAB CBA


3 2 1=6
45. How many two-digit prime numbers have a remainder of 2
when divided by 7?
 
(A)none
(B)one
(C) two
(D)three
(E)more than three
 
45. D
The list is 16, 23, 30, 37, 44, 51, 58, 65, 72, 79, 86,93.
The only primes less than 10 are 2, 3, 5, and 7.
Now, 16, 30, 44, 58, 72 and 86 are even, and 51 and 93 are
divisible by 3 and 65 is divisible by 5 so the only possible
primes are 23, 37, and 79. Since none of these is divisible
by 7, they all are prime. The correct choice is (D).

You might also like